Composite values of shifted exponentials
Abstract.
A well-known open problem asks to show that is composite for almost all values of . This was proposed by Gil Kalai as a possible Polymath project, and was first posed by Christopher Hooley. We settle this problem assuming GRH and a form of the pair correlation conjecture. We in fact do not need the full power of the pair correlation conjecture, and it suffices to assume an inequality of Brun–Titchmarsh type in number fields that is implied by the pair correlation conjecture. Our methods apply in fact to any shifted exponential sequence of the form and show that, under the same assumptions, such numbers are -almost primes for a density of natural numbers . Furthermore, under the same assumptions we show that is composite for almost all primes whenever .
2010 Mathematics Subject Classification:
11A41, 11R44, 11R421 Introduction
It is a notorious open question to determine whether a shifted exponential sequence for given and produces infinitely many primes as ranges over the positive integers . One expects that this should be the case whenever there is no obvious reason for to be composite for all large , the obvious reasons being that either has a fixed prime divisor or that it factors as the result of a polynomial identity. This leads to the following question.
Question 1.1.
Let and be integers. Assume that for every there exists such that:
-
(i)
The sequence has no fixed prime divisor;
-
(ii)
There is no , such that is an th power of a rational number, and further if there is no such that );
Then does the sequence contain infinitely many primes?
Here case (ii) corresponds to the well-known fact that the binomial is reducible if and only if is an th power of a rational number for some , , or and for some rational . The exclusion of stems from the fact that for one easily sees that the only possible primes in the sequence are of the form , and as is discussed below, probabilistic heuristics suggest that there are only finitely many such primes (even though (i) and (ii) hold for , say). See Section 11 for further discussion on the necessity of the conditions.
Question 1.1 is closely connected with two conjectures that are among the oldest in number theory: the existence of Mersenne primes and Fermat primes. Mersenne primes are primes of the form with a prime; one easily sees that any prime of the form must be of this form. Many of the largest known primes are Mersenne primes. Fermat primes are primes in the sequence ; again, all the primes of the form are easily seen to have this shape. The first five Fermat numbers () are all prime, but extensive numerical searches have produced no further Fermat primes. A widely-believed conjecture (supported by probabilistic arguments; see [11, Problem A3]) is that there are infinitely many Mersenne primes but only finitely many Fermat primes. These two assertions seem to be well beyond reach of all known methods.
Although little is known about the primality of , it is noteworthy that can be prime only for a set of integers of natural density ; this follows immediately from the form that the exponents of the Mersenne and Fermat primes take. In light of this, one conjectures more generally that is prime for a natural density of natural numbers . Even this is still an open problem. The problem, in the concrete special case of the sequence , was suggested by Gil Kalai in [16] as a possible Polymath project (with the comment that it “might be too hard”). The problem was originally studied by Christopher Hooley in his book [13], and was popularized by Peter Sarnak111Sarnak remarks in [30] that “Even a problem like being composite for almost all is very problematic (Hooley).” [30].
In the discussion in [16], it was suggested that in order to make any progress on the compositeness of one should assume GRH. Our first main result confirms that is indeed composite for almost all , as well as the analogous result for general shifted exponential sequences , assuming GRH and an additional Brun-Titchmarsh type inequality for primes satisfying conditions of form “, is a perfect th power modulo ”. We postpone detailed discussion of the hypotheses to Subsection 1.1.
Hypothesis 1.2 (A Brun–Titchmarsh estimate on average).
Let and be fixed coprime integers with . The following holds for any small enough :
Let be a positive integer and let be a prime with and . Then, uniformly for , we have
(1.1) |
We then state our main results.
Theorem 1.3.
Assume GRH and Hypothesis 1.2. Let and be integers. The natural density of positive integers such that is composite is .
Remark 1.4.
Hooley had shown in [13] that is composite for almost all assuming GRH and an essentially self-serving hypothesis. Our task in this paper is to remove this self-serving hypothesis and replace it with a more natural one.
Remark 1.5.
One motivation for Sarnak to popularize Hooley’s problem was that Bourgain, Gamburd and Sarnak [3, Theorem 3] managed to settle the analogous problem for the Markoff numbers. Markoff triples are defined as the solutions to the Diophantine equation , and the Markoff numbers are the increasing sequence formed by the largest coordinates of Markoff triples (with multiplicities). By the result of [3] almost all Markoff numbers are composite, and their number up to is . Therefore, the sequences are even sparser.
Sarnak [30] also connected Hooley’s problem to the affine sieve developed in [29], [2]; the affine sieve theorem of Salehi Golsefidy and Sarnak [29] applies to counting almost primes lying in an orbit of a group of affine linear transformations under the assumption that the Zariski closure of the group is Levi-semisimple. The authors of [29] present a heuristic argument for the necessity of the Levi-semisimplicity condition, which boils down to understanding almost primality questions for shifted exponential functions, such as the one above.
In addition to showing compositeness of for almost all , we are more generally able to show that is not a -almost prime (a number with at most prime factors) for a density set of natural numbers . We prove this in the following strong sense.
Theorem 1.6.
Assume GRH and Hypothesis 1.2. Let and be coprime integers. Then there exists a constant such that the natural density of positive integers such that is .
Here, as usual, denotes the number of distinct prime factors of .
We will in fact prove that the number of prime divisors of which are less than is almost always . On the other hand, should of course have a lot of prime factors as well, but our method does not apply to detecting these large factors.
One naturally wonders whether one can say something about the compositeness of even when is restricted to an interesting subset of natural numbers, particularly the primes. In the case of Mersenne primes, which are of the form with prime, it appears unknown that there are even infinitely many composite numbers in this sequence. Nevertheless, we are able to apply our methods also to the sequence with prime exponents, as long as we are not in the case , that corresponds to Mersenne primes.
Theorem 1.7.
Assume GRH and Hypothesis 1.2. Let and be integers with . The relative density of primes for which is composite is .
1.1 The hypotheses
In this subsection we discuss the hypotheses appearing in our results, namely the generalized Riemann hypothesis (GRH) and Hypothesis 1.2. We also connect Hypothesis 1.2 to the pair correlation conjecture (PCC) stated below.
The precise form of the generalized Riemann hypothesis (GRH) that we need is as follows. It only involves certain special field extensions, which we call Kummer-type extensions.
Definition 1.8.
We say that a field extension is a Kummer-type extension if is of the form , where is a primitive root of unity of some order , are integers and for all .
GRH: For any Artin -function222See [24] for an introduction to the theory of Artin -functions. associated with a Kummer-type extension, its zeros in the critical strip all lie on the critical line .
We then give a couple of remarks on Hypothesis 1.2.
-
•
The condition “, is perfect th power mod ” is naturally viewed as “ splits completely in ”. Hence Hypothesis 1.2 asks for a bound on the number of totally split primes in a number field.
-
•
One may view the hypothesis as a generalization of the classical Brun–Titchmarsh inequality (with extra averaging), which states that, uniformly in the region , we have
This, combined with the fact that can only split completely in extensions of the form , gives an upper bound for the left-hand side of (1.1) of the form , and so this trivial bound falls short of Hypothesis 1.2 by a few logarithms at most.
-
•
One would more strongly expect for the left-hand side of (1.1) a bound of
(1.2) (and also without the primality assumption on ), since one can prove under our assumptions that , and the probability of a prime splitting completely in should be comparable to with a wide range of uniformity, as is known in the case of cyclic extensions.
-
•
Hypothesis 1.2 should be true in the full range ,. However, as is clear from the bound given by the Chebotarev density theorem, (1.1) becomes much more challenging to prove as the quantity increases. Therefore, in our arguments we exercise great care to minimize the range of in which we assume (1.1), in the hope that the case of small would turn out to lie less deep than the case of large .
-
•
The bound (1.1) resembles a Brun–Titchmarsh estimate in number fields, as our naming of the hypothesis suggests. Unfortunately, the current Brun–Titchmarsh analogues of the Chebotarev density theorem are too weak to imply the Hypothesis. For example, the results of [32] or [5] are not applicable when .
Next we connect Hypothesis 1.2 to the pair correlation conjecture (PCC), which in fact implies our hypothesis.
PCC: Let be a Kummer-type extension, and let be the Artin -function associated with an irreducible character of . Define the pair correlation function
where run through the imaginary parts of zeros of on the critical line and is a weight function (note that is always real-valued). Further define the conductor , where , where is the degree of the field extension and is the Artin conductor of . With this notation, for any and , we have
Remarks.
-
•
This PCC conjecture is denoted by M. R. Murty, V. K. Murty and Wong in [21, Conjecture 3.2] and is a special case of a more general conjecture stated there. They state that this conjecture first arose in unpublished work of M. R. Murty and V. K. Murty 20 years earlier.
-
•
Evidence for PCC includes the result [21, Proposition 3.1], which unconditionally gives
so the content of the conjecture is in reducing the power of logarithm here. As noted in [21], the form of PCC stated here is weaker than some other forms of the pair correlation conjecture in the aspect that they further require an asymptotic formula for . In particular, Montgomery’s [19] classical pair correlation conjecture for the Riemann zeta function (which corresponds to and ) predicts not only an upper bound of for the pair correlation function of but more strongly an asymptotic formula of the same order of magnitude. Montgomery in fact proved this asymptotic formula unconditionally when in the notation of the PCC conjecture above. See also [25] for numerical evidence for the pair correlation conjecture in the case of the Riemann zeta function, [28] for an asymptotic formulation for all automorphic -functions, and [22] for the formulation for any Dirichlet series in the Selberg class.
Theorem 1.9.
PCC for all Kummer-type extensions implies Hypothesis 1.2.
In fact, as will be clear from the proof, PCC implies a much stronger version of Hypothesis 1.2, where we can take .
Let us briefly comment on how our two assumptions, GRH and Hypothesis 1.2, enter the proof. The GRH assumption is clearly necessary, as will be seen from formula (1.4) below (which is currently provable under GRH but not unconditionally). Also, Chebotarev’s density theorem is a key tool in our proofs, and in order for its error bounds to be strong enough we need to assume GRH.
Hypothesis 1.2 is put into use in only one part of the proof. The idea is roughly as follows. For a prime, let denote the least positive integer such that (if such an integer exists). In the course of the proof, we need upper bounds for the number of primes that satisfy and , for just a bit larger than (say or ). This condition can be naturally interpreted in terms of splitting of primes in certain Kummer-type extensions, and the Chebotarev density theorem tells us that such are roughly333One does not necessarily have exact equidistribution (in sense of the natural density) even for fixed. This technical detail is discussed in more detail later on. equidistributed among the different values of , at least for large in terms of . The small values of (compared to ) are more difficult. However, as we only need upper bounds rather than actual equidistribution, we can afford to weaken the condition to for being a relatively small divisor of the modulus. Now, Hypothesis 1.2 essentially states that the values for the primes with are not clustered into a too small a set of residues, which is what we need in the proof.
In the case is very close to (), we may neglect the condition and merely work with the congruence conditions . Such a crude estimate does not provide estimates good enough for our purposes in regions larger than , so for larger values of one has to somehow account for the condition . As Hypothesis 1.2 is essentially our only tool for such considerations for relatively close to , we require it to be applicable in the region .
1.2 Unconditional work
Not much is known about the composite values of unconditionally (except in the trivial case where conditions (i) and (ii) in Question 1.1 are not satisfied). Note that if the congruence has a solution (which we assume to be the minimal positive solution), the general solution to the congruence is given by . The set of such has natural density . Thus, by the Borel–Cantelli lemma, a necessary (but not sufficient) condition for almost all numbers of the form to be have a prime factor for every given is that
(1.4) |
This estimate has not been proved unconditionally, meaning that if one wants to make progress on the composite values of , one must assume a conjecture that implies (1.4). It seems that the best known unconditional estimate for the number of primes dividing some element of the sequence is (in contrast with the conjectured order of magnitude of ), proved by M. R. Murty, Séguin and Stewart [23]; this is a bound that is unfortunately far too weak to imply (1.4).
It is also worth noting that Hooley showed unconditionally in [13] that the sequence produces primes (the Cullen primes) for a density zero of natural numbers ; as pointed out by Elsholtz in [6], this is a lot easier than the corresponding question for the sequence , since it is easier to control the distribution of Cullen primes in residue classes (regarding the problem of the sequence , he states that “current methods do not work” for it). Hooley’s method was further refined by Rieger [26] to yield that is prime for a relative density of primes .
1.3 Connection with the Artin primitive root conjecture
The most natural conjecture known to imply (1.4) is GRH; this implication follows from work of Moree and Stevenhagen [20]. Their work is related to Hooley’s [12] work on Artin’s primitive root conjecture, which is the statement that for infinitely many primes, whenever is a fixed integer that is not a perfect power. A wide generalization of Artin’s conjecture was proved by Lenstra [18], again under GRH, and in fact our proof of Theorem 1.3 involves (among other things) adapting his work to sequences of the form . This produces the following intermediate result containing (1.4) (by Mertens’ theorem), which may be of independent interest.
Corollary 1.10.
Assume GRH. Let be an integer that is not a perfect power, and let . Let be the largest integer for which is a perfect th power (if , we define ). Then the set of primes satisfying for some and possesses a natural density, which is positive and can be computed explicitly.
1.4 Acknowledgments
The authors thank the anonymous referees for numerous helpful corrections. The authors thank Peter Sarnak and Jesse Thorner for helpful discussions. The first author was supported by the Emil Aaltonen foundation and worked in the Finnish Centre of Excellence in Randomness and Structures (Academy of Finland grant no. 346307). The second author was supported by a Titchmarsh Fellowship and by European Union’s Horizon Europe research and innovation programme under Marie Skłodowska-Curie grant agreement No 101058904.
2 Notation, conventions, and some preliminaries
Without further mention, we assume GRH in the lemmas and theorems that follow. We will assume Hypothesis 1.2 only in the proof of Lemma 7.1 below.
We may assume in the proofs of Theorems 1.3 and 1.6 that is not a perfect power, as the case of being a perfect power immediately reduces to the generic case. If , the conclusion of Theorems 1.3 and 1.7 is trivial, since for and for and odd. Thus we may assume when proving these theorems. We may also assume that , since otherwise is composite.
If is a prime not dividing , we denote by the least positive integer such that .
We denote by any primitive th root of unity.
We let denote the divisor and Euler phi functions, respectively. We let stand for the greatest common divisor of and and for their least common multiple. By we denote the product of the prime factors of .
The natural density of a set is defined as the limit
provided that the limit exists. We similarly define the relative density of a set in the set as
assuming that and that the limit exists. The density of a subset of the primes is to be considered as its relative density in the set of primes.
For any subset of the primes, we define .
Given an extension of number fields, a prime of , and a prime of lying above , we define the Artin symbol as the unique element satisfying
for all . Further, if and is a rational prime unramified in , we define as the conjugacy class of possible values of with lying above ; it can be checked that such values do indeed form a conjugacy class. (This definition could be extended to as well, but we will only need the case .)
The prime is said to split completely in if is the conjugacy class consisting of the identity of .
We use the following version of the Chebotarev density theorem, due to Serre [31] and conditional on GRH (this improves on work of Lagarias and Odlyzko [17]).
Lemma 2.1 (Chebotarev density theorem).
Assume GRH. Let be a finite Galois extension of with Galois group . Let be a conjugacy class of . The number of (rational) unramified primes with and Artin symbol is
where is the discriminant of and .
Proof.
This is [31, Théorème 4]. ∎
3 Overview of the method
In this section, we give a sketch of the proof method of Theorem 1.3; the actual details in subsequent sections are slightly different and more complicated, but the purpose of this section is just to illustrate the approach.
As already mentioned, we may assume that is not a perfect power. Let
(3.1) |
since contains those primes for which is a primitive root, under GRH the set contains a positive proportion of the primes. For each , there exists a unique integer such that . Now, for any integer , we have , implying that is not prime (except possibly for ). The goal is to prove that the density of integers covered by such arithmetic progressions is . Thus the statement is that the residue classes for are nearly (up to a vanishing proportion of numbers) a covering system of the integers. This is generally speaking a rather delicate condition; see [8] and [1] for work on finite sets of congruences covering all but an -proportion of the integers444Note that whether or not residue classes cover almost all numbers depends heavily on the values of the ; for example, it is known that the proportion of integers covered by for approaches as . Also if is any subset of the primes with sum of reciprocals less than , say, then for leaves uncovered a positive proportion of all numbers..
If the conditions were independent of each other (which would be the case by the Chinese reminder theorem if the moduli and were coprime for all ), the density of integers not covered by such congruence conditions would be
Under GRH the relative density of inside the primes is positive, so by Mertens’ theorem the above product evaluates to .
However, since and are typically not coprime555Note that it does not help to restrict to a subset of primes for which are pairwise coprime, since such a set always has finite sum of reciprocals. If for example we restrict to primes such that is also prime, then it is known that such primes have bounded sum of reciprocals (even though their infinitude is not known)., we have to take into account the dependencies between the conditions imposed by different primes. For this we utilize the second moment method.
Lemma 3.1.
Let be any finitely additive probability space, and let be events. Denote
(3.2) |
Then, for any , we have
(3.3) |
Proof.
This follows by writing the left-hand side as
and using Chebychev’s inequality to upper-bound this as
and then expanding out the square. ∎
Remark 3.2.
In our case, the events correspond to a “random” integer being congruent to , where ranges over , from which we get . (See Section 5 for a precise formulation of this idea.) Then the expected value appearing in (3.2) is, more or less, equal to
(3.4) |
For reasons explained below, we will in fact restrict to a subset of where , which moreover has a positive relative density inside the primes. This then allows for an asymptotic evaluation of (3.4). We now turn to the sum of the pairwise intersections in (3.3), which are much more difficult to analyze.
Let and be primes, and write . The system has a solution if and only if . If a solution exists, then , and otherwise . Thus
(3.5) |
If the pairs were equidistributed modulo all , even when conditioned on the event , we would see that and are independent “on average”, resulting in the same asymptotics for (3.5) as for (3.4).
Unfortunately, the set does not have the required equidistribution property for all moduli.666For example, if and , then must be even. More generally, congruence conditions for give information on quadratic residues modulo , leading to bias even in power-free cases. For example, if and , , then is a quadratic residue and is not, so must be even. In Section 4, we construct a positive density set of primes (depending on ) for which is solvable and for which the smallest solution of the congruence does enjoy the required equidistribution property for all fixed by adapting the method of Lenstra [18]. We then apply Lemma 3.1 to this subset of primes in place of (in which case showing the existence and positivity of requires some work).
The main difficulty then is that we need to estimate (3.5) in all ranges of , not only for fixed (even though the case of bounded should give the main term in (3.5) and larger should only contribute an error term). This will be achieved in four steps, carried out in different sections.
- (i)
- (ii)
-
(iii)
The case of very large (say ) is handled (unconditionally) by applying Selberg’s sieve in Subsection 7.1. The range here is optimal in the sense that this argument (which starts by dropping the condition ) would not work in any larger regime.
- (iv)
Combining the estimates for these four different regimes will yield Theorem 1.3.
We comment on some difficulties arising with controlling the contribution of large values of . To bound the sum in (3.5) one has to bound the number of primes (or more generally ) satisfying summed over all . (The contribution of large would be too large if the values attained only a few values, so we cannot just drop the condition on and work only with the congruence conditions .) This condition can be naturally interpreted as splitting completely in a certain field extension, at least for , where is the set constructed in Section 4.
If is small (say ), one can apply the GRH-conditional Chebotarev density theorem for all separately (see case (ii) above). If is of magnitude , one can barely apply the Chebotarev density theorem for a single value of directly, and to control all values of one would need a uniform error term. If is much larger than , say , the GRH-conditional square root error term is inapplicable. It is here that we use Hypothesis 1.2.
Note that while for fixed (see case (i)) we need exact (asymptotic) equidistribution, for large values of we only need that the residues do not obtain a small set of values very often. Heuristically each value occurs roughly of the time (even with quite large), but we only need that each value occurs at most of the time. This is indeed what Hypothesis 1.2 tells us.
4 Equidistribution of the discrete logarithm
Let be the largest integer such that is a perfect power of order . Recall from Section 2 that we can assume is not a perfect power. Let (which depends on and ) be a set of primes defined as
Note that for any we have , so the congruence has a solution.
Let be the smallest nonnegative solution to for . For each , let
(4.1) |
Note then that for any solution to satisfies . Note also that
Indeed, for the forward implication note that if and , then is equivalent to , and here is a th power . Conversely, suppose that , and . Then by the assumption on we have for some , so for some , and this implies .
Our goal in this section is to prove the following two lemmas.
Lemma 4.1.
The relative density exists and is positive.
Lemma 4.2.
For and fixed, the relative density exists and does not depend on .
Before proving these lemmas, we make some preliminary considerations.
We build on the arguments of Lenstra in [18]. Let
Clearly if , then . Since is a perfect th power, (indeed, if , then ). Furthermore, is either or for some . In either case,
For a prime , define
to be the smallest power of which does not divide , and let
We can now characterize the set as follows.
Lemma 4.3.
A large enough prime satisfies if and only if splits completely in but not in any of the .
Proof.
We recall a few simple facts from Galois theory. Firstly, by Dedekind’s factorization theorem, an unramified prime splits completely in a field extension if and only if the minimal polynomial of factorizes into distinct linear factors for large enough. Secondly, splits completely in if and only if splits completely in both and . Thirdly, the cyclotomic polynomial factorizes into distinct linear factors if and only if .
From these three facts and the well-known fact that the compositum of Galois extensions is Galois, we see that a large prime splits completely in but not in any of the if and only if is a th power , is a th power , and is not a th power for any . The first and last condition are equivalent to , and together with the second and third condition this is equivalent to , so the claim follows (note that the special case gives a characterization of the set ). ∎
For a squarefree integer, let be the compositum of for primes and let
Define
One would now expect that
(4.2) |
where is the product of the first primes. This is because is by the Chebotarev density theorem equal to the density of primes which split completely in but in none of . Formula (4.2) was indeed proved by Lenstra [18, Theorem 3.1] under GRH. Thus, in particular, the density exists.
Another relevant result of Lenstra concerns whether equals to or not. By (4.2), this corresponds to understanding the behavior of the local densities . Clearly for , so if for some , then . In fact, the converse is true as well, as shown by Lenstra [18, Theorem 4.1]: if for all , then is strictly positive. The idea of Lenstra’s proof is that the conditions of not splitting completely in are independent of each other and of the condition of splitting completely in , except for some finite set of primes . In other words, and are linearly disjoint extensions for and large enough. This allows one to write the density in (4.2) as an infinite product
(4.3) |
where is the product of exceptional primes and the terms correspond to the density of primes not splitting completely in (see [18, (5.11)]). Note that the infinite product in equation (4.3) converges to a strictly positive value, therefore giving both a method for determining whether or not and a method for calculating this density.
We are now ready to prove our lemmas.
Proof of Lemma 4.1.
Consider the case above. By (4.3), it is sufficient to check that for all . Fix and consider the field
Let be the automorphism which fixes , and , and which maps to for all . Assuming that this is well-defined, the result follows from the Chebotarev density theorem (since then fixes but none of ). To prove that there exists such a map , let . It suffices to prove that extending by the elements one by one always increases the degree of by a factor of (note that the degree increases by at most a factor of ). In other words, it suffices to prove that the degree of over is .
To prove this we use the following simple observation (see [15, Proposition 3.5]): Let be a positive integer which is not a perfect power. For positive integers and with , the degree of is either or , where the latter case occurs if and only if .777The proof is by basic Galois theory. See (4.5) below for the proof of a closely related statement. This result will be used subsequently without further mention.
We now have
where by the above observation the degrees in the numerator and denominator are “what one would expect”, namely and , respectively. Here dividing by accounts for the fact that . This concludes the proof of Lemma 4.1. ∎
Proof of Lemma 4.2.
We then prove Lemma 4.2. Fix and , and let be the product of exceptional primes over all so that (4.3) is valid for all . Let be as in (4.3). Recall that is equal to the density of elements fixing but not for any . One may thus write, by inclusion-exclusion,
(4.4) |
We now turn to proving that, for any fixed , does not depend on . This follows from the following more general claim: Let and be positive integers satisfying and . We have
(4.5) |
In particular the degrees of the extensions do not depend on .
Note that the degree in (4.5) is at most . Indeed,
The first term equals . The second one is at most , as . The third term is similarly at most , as , since .
We then prove that this bound is tight. We claim that the numbers
where and , are linearly independent over , which gives the lower bound for the degree implicit in (4.5). By [10] it suffices to check that
(4.6) |
for any such and not both . Suppose that (4.6) fails. Let
Now is an integer for which . Thus, arguing as before, must be a perfect th power of a rational number. Since and are coprime, this means that
and
where we used the fact that is not a perfect power and is an th power. The latter condition gives , and since , we have . Now the first condition gives , so we must have . This concludes the proof of Lemma 4.2. ∎
5 Applying the second moment method
As before, let be the smallest positive integer for which (whenever it exists).
Let the set be as constructed in Section 4. For , let
and let denote the uniform probability measure on . Also denote . By Lemma 3.1, in order to prove Theorem 1.3 it suffices to prove the estimate
(5.1) |
By Lemma 3.1, the quotient above is always , so it suffices to establish an upper bound of . Recalling that for , the denominator is
(5.2) |
by Lemma 4.1 and partial summation.
The numerator can be written as
Note that . Let be a large, fixed integer. By Möbius inversion, we write
The problem thus reduces to considering sums of the form
for various .
Define
and
Now we have
In Sections 6 and 7, we will prove that for any and for (where is a fast enough growing function of ) we have
(5.3) |
and
(5.4) |
Combining (5.2), (5.3) and (5.4) and letting gives a bound of for the right-hand side of (5.1), and letting slowly in terms of gives Theorem 1.3. Thus, our remaining task in the proof of Theorem 1.3 is proving (5.3) and (5.4).
6 Contribution of small gcd
In this section, we prove the desired estimate (5.3). We handle the cases and separately.
6.1 Case 1.
Let . By partial summation, we have
The first term here is . The second term can be bounded by splitting the integration interval into parts and , where is a fast growing function of such that
uniformly for ; such a function exists by Lemma 4.2. We then have
and
All in all, we have
Squaring gives
Summing over and using Lemma 4.2 (which says that is independent of ), we get
(6.1) |
We multiply this by and sum over and with . For large enough in terms of , this gives
where we used
6.2 Case 2.
Now we bound the contribution to (5.3) from the terms . Crude estimation gives
We split the summation over into the intervals and . Applying the Brun–Titchmarsh inequality for and a trivial estimate for gives
Since implies , we can bound the contribution to (5.3) from the terms by
which is the desired bound.
7 Contribution of large gcd
We are now left with the task of proving (5.4). Define
Furthermore, let be the set of primes for which and for which is not a th power for any prime . We now see that , where these larger sets and are easier to control.
We estimate
Call the expression inside the sum . We divide the sum into three parts depending on the relative size of and as follows. Set . We have
where
Denote the sums on the right by , , , respectively. In the following subsections we prove that the sum of over all and is small enough for each .
7.1 Bounds for very large gcd
We begin by estimating on average over and . This sum will be dealt with using Selberg’s sieve, and in particular this part of the proof is unconditional. We have
(7.1) |
We claim that for any , for and , we have
(7.2) |
Once this has been shown, multiplying by and summing over and dyadically we can upper bound the contribution of by
by our choice . This is certainly .
To prove (7.2), we first note that the contribution of the terms to that sum is , so it suffices to prove a bound of for the off-diagonal contribution. Also observe that if , then for some . Thus, the contribution of to (7.2) is
By Selberg’s sieve, in the form of [14, Theorem 6.4] (see also formulas (6.83)–(6.86) there), this is further bounded by
(7.3) |
Note that by Cauchy–Schwarz for any we have
(7.4) |
Therefore, (7.3) is at most
as wanted.
7.2 Bounds for medium gcd
We then deal with the sums for which we assume GRH (but not Hypothesis 1.2). We divide into dyadic intervals to obtain
(7.5) |
Note then that implies and that is an th power , so by Galois theory splits completely in . Thus the Artin symbol is equal to .
Now we are in a position to apply the Chebotarev density theorem (Lemma 2.1). Under GRH, it gives
where is any conjugacy class of . In the case of the specific extension above, we easily see that (see Lemma 8.1), and also by [4, Lemma 1.2] we have , so if is the identity of , we get
Therefore, (7.5) is
Summing this over all , multiplying by and summing over gives
This is a good enough estimate.
7.3 Bounds for large gcd
We then turn to estimating on average over and . This is the only part of our proof where Hypothesis 1.2 is used.
Let
(7.6) |
where is small but fixed and is chosen to be small enough in terms of . For each we let denote such a divisor of .
We wish to estimate the quantity
(7.7) |
We split
(7.8) |
We now present a general bound for the contribution of a single to the first sum in (7.3). This is the only part of the paper where Hypothesis 1.2 is put into use.
Lemma 7.1.
Assume Hypothesis 1.2. Let , and let and with and . Then, provided that , we have
Proof.
Note first that if , then . Observe also that
(7.9) |
for any : if , then is a perfect th power and thus must be a th power for some prime , implying . Also note the trivial inequality
(7.10) |
Using (7.9) and (7.10), we obtain
where is the set of primes which split completely in but which do not split completely in for any . In particular,
Hence, under Hypothesis 1.2, we have
which produces the assertion of the lemma. ∎
Splitting dyadically over and , we see that
(7.11) |
Regarding the first term in (7.3), if we multiply it by and summing over that are powers of two, we get a bound of
which is certainly small enough.
Now, to conclude the proof of Theorem 1.3, it suffices to prove the following two claims.
Lemma 7.2.
We have
(7.12) |
Lemma 7.3.
We have
(7.13) |
Proof of Lemma 7.2.
Analogously to (7.1), we first make for the crude estimate
Note that applying the Brun–Titchmarsh inequality to this would cost us some nontrivial factors, since that inequality involves savings of rather than if . Therefore, we instead exploit the summation over . Let us restrict to , and ; we will later sum dyadically over different . Let
and observe that . Our task is thus to upper bound
(7.14) |
Let us first estimate . Let . Note that
where is the part of the sum with and is the complementary part. By Mertens’ theorem, we have
(7.15) |
When it comes to , we crudely estimate
Hence,
(7.16) |
Next, note that by (7.16) the diagonal contribution to (7.14) (crudely dropping the condition that ) is
and it only appears if . Multiplying by and summing dyadically over and , we get a contribution of for the diagonal terms. Since for sufficiently fast decaying , this is . We may henceforth restrict to in (7.14).
We are then left with upper bounding the non-diagonal contribution in (7.14). To this end, we will apply Selberg’s sieve to the set
We claim that if is squarefree, and , then
(7.17) |
where satisfies a level of distribution estimate
(7.18) |
and is the completely multiplicative function given at primes by
We note that for the condition is equivalent to or , and the residue classes and are distinct from each other and from whenever . Therefore, in order to prove (7.18) it suffices to prove that if , then for coprime to we have
where
(7.19) |
and is the completely multiplicative function given by
For proving (7.19), we wish to split as a bilinear sum and then apply a general Bombieri–Vinogradov type result for such objects.
Observe the decomposition
where
The contribution of to (7.19) is negligible (similarly as in (7.15)), whereas the part has level of distribution by a variant of the Bombieri–Vinogradov theorem (see [14, Theorem 17.4], where one just needs to verify the Siegel–Walfisz condition for . This condition can be proved for similarly as for the indicator of the primes). We claim that also the part has level of distribution . Since is supported on , we see that if has level of distribution , so does . For the sequence , we indeed obtain level of distribution by noting that , and here has level of distribution by the classical Bombieri–Vinogradov theorem, whereas each of the terms has level of distribution by [14, Theorem 17.4].
Now we have
and (7.19) follows from this by Cauchy–Schwarz and the divisor sum estimate and the trivial bound .
Proof of Lemma 7.3.
We split the sum in (7.13) into two parts: those with and the remaining . The contribution of the former part is
(7.20) |
which is a strong enough bound.
For the contribution of the latter part, note that all counted by it belong to the set
Therefore, their contribution is
and the same argument as in the proof of Lemma 7.2 gives
so the claim follows. ∎
8 Proof of Hypothesis 1.2 under the pair correlation conjecture
In this section, we prove Theorem 1.9 to the effect that PCC implies Hypothesis 1.2. We begin with a few preliminary lemmas. Note that in Hypothesis 1.2 the parameter is prime, but that hypothesis will only be used in (8.5) and (8.6) below (and it would suffice to assume that does not have abnormally many prime factors), and the lemmas before the proof do not assume to be a prime.
8.1 Lemmas on Kummer-type extensions
We first present a standard lemma (see e.g. [15, Proposition 3.10]).
Lemma 8.1.
Let and be multiplicatively independent integers. We have
and
for all positive integers , and .
In what follows, we denote
where . We bound the number of conjugacy classes in .
Lemma 8.2.
For each , the number of conjugacy classes of which contain elements fixing is at most . Furthermore, the size of each such conjugacy class is at most .
Proof.
To each element , we associate a triple such that , and . We write . Consider the set of such tuples with a binary operation given by
This operation is clearly associative. The identity of is . The inverse of under is , where is the inverse of modulo . Thus the set of these triples with operation forms a group. The conjugacy class containing is given by the set of elements of the form
where varies.
Let be an element of fixing . Now and . Consider for variable the conjugation
Since varies, we see that there are at most conjugacy classes corresponding to elements fixing , one for each integer such that there exists a map satisfying . The sum of the sizes of these conjugacy classes is at most , so each single class is of size at most . ∎
Note that the proof of Lemma 8.2 gives that if some element of a conjugacy class of fixes , then all of the elements of it do.
We also need a bound for the number of conjugacy classes in .
Lemma 8.3.
The number of conjugacy classes in is .
Proof.
Recall the notation of the proof of Lemma 8.2. Let be the set of all triplets of the form equipped with the operation , so is a subgroup of . By Lemma 8.1 the index of in is bounded. This implies by [9] that the number of conjugacy classes of is at most a constant times that of . We therefore consider only the group from now on. In what follows, we identify the elements of with integers in .
For each let and let be the set of elements with and . Let be the union of over all . We claim that each element in is conjugate to at least one element in .
Let be given. As in the proof of Lemma 8.2, the conjugacy class of consists of elements of the form
There exists an element such that divides . Fix such an element . We may now choose and such that divides and belongs to . For this choice of and we have , proving the claim.
The result follows by
∎
Lemma 8.4.
Let be the conjugacy classes of which fix but which do not fix any of the fields , where ranges over the prime divisors of . Then the are pairwise disjoint.
Proof.
Assume that for some . Thus, fixes both
and
In particular, fixes . Let and let be such that . We see that fixes
where is some rational number, and thus fixes .
Since fixes both and , we see similarly as above that also fixes . Noting that is a proper divisor of , we deduce that fixes for some prime . Finally, note that fixes . Thus, fixes , which is a contradiction with the definition of . ∎
8.2 A Chebotarev-type estimate in the mean square
Having established these preliminary lemmas we proceed with deriving the implication of PCC needed. To begin we need a further conjecture, the Artin conjecture (AC) on -functions.
AC: The the Artin -functions associated with a field extension are analytic in the whole complex plane.
Although Artin’s conjecture remains open in general, we will see in a moment that it can be proved for fields that occur in our setting, so it is not a restricting assumption.
Lemma 8.5 (Murty–Murty–Wong).
Let be a field extension whose Artin -functions satisfy GRH, AC, and PCC. For a conjugacy class of , define the Chebychev function
Then we have
(8.1) |
where is the set of rational primes that are ramified in and is the set of irreducible representations of .
Proof.
This is [21, Theorem 5.1] with the choices , , and that correspond to our assumptions. ∎
Although Artin’s conjecture remains unsolved for general -functions, the next lemma says that AC holds for all Kummer-type extensions.
Lemma 8.6.
Hypothesis AC holds whenever is a Kummer-type extension.
Proof.
It is known (see [34]) that Artin’s conjecture holds for the field extension whenever its Galois group is metabelian, meaning that it has a normal subgroup such that both and are Abelian. Thus, for any given Kummer-type extension field , it suffices to find a subfield such that and are both Abelian extensions and is normal in . We choose . Then is cyclic, so certainly Abelian. To show that is also Abelian, it suffices to show that each is Abelian, since the compositum of Abelian extensions is also Abelian. Again, is a cyclic extension, and therefore Abelian. What remains to be shown then is that is a normal subgroup of . As in the proof of Lemma 8.2, for each we write
where this tuple is uniquely determined by the conditions and for all . The relation is an equivalence relation on . If is arbitrary, we may write for some . Now , so . Since , are arbitrary, the claim follows. ∎
Proposition 8.7.
Assume GRH and PCC. For , we have
Proof.
It is well known that equals to the number of conjugacy classes in , which by Lemma 8.3 is . Further, we have by Lemma 8.1. Lastly, note that if , then and that by [4, page 490] we have
for some . A computation of the discriminants shows that
so
In conclusion, we have the bound
(8.2) |
To pass from to , we use partial summation. Note that
so by partial summation
This gives
and further
(8.3) |
By Cauchy–Schwarz, we can bound
(8.4) |
Now, multiplying (8.3) by , summing over and using (8.4), (8.2), we get
as desired. ∎
We are now ready to prove Hypothesis 1.2 under PCC.
Proof of Theorem 1.9 assuming PCC.
We may assume that we are in the regime , say, since otherwise the GRH-conditional estimate in (1.3) is good enough.
Recall that are the conjugacy classes of which fix but which do not fix any of the fields , where ranges over the prime divisors of . By Lemma 8.2, we have and for all and . Let denote the number of primes such that the Artin symbol of with respect to is . We now have
By Lemmas 8.1 and 8.2, and the assumption that is prime, the latter sum is
(8.5) |
When it comes to the first sum, we estimate using Cauchy–Schwarz that
9 Almost prime values of shifted exponentials
In this section, we prove Theorem 1.6. The proof follows along the same lines as the proof of Theorem 1.3.
Proof of Theorem 1.6.
Firstly, we deal with the simple cases . The case is trivial. For the cases , we observe that by the Hardy–Ramanujan theorem almost all have prime divisors, and if is odd, then for either choice of the sign . To conclude, we apply a theorem of Zsigmondy [27], according to which each term in the sequence for (and also for with a few exceptions) has a primitive divisor, that is, a prime factor that does not divide for any .
10 The case of prime exponents
We turn to the proof of Theorem 1.7. Most steps go through similarly as in the case of Theorem 1.3, but there are a few extra complications, namely the set from Section 4 needs to be defined differently, using the “-trick”, and therefore we will also need Lemma 10.1 below.
Proof of Theorem 1.7.
Let and be the largest integers such that (respectively ) is a perfect power888Note that here we can no longer assume that is not a perfect power. of order (respectively ). Define . Let , where is a large parameter. We redefine the set in Section 4 as
As before, define by (4.1). We observe then that for we have . Indeed, if this did not hold, we would have
which is absurd as .
We begin by proving that exists and is positive, and furthermore that is independent of as long as is coprime to . We remark that Lenstra’s results in [18] do not immediately apply to our situation. In [18], the results concern the order of the reduction of a single multiplicative subgroup of modulo primes, while in our case we wish to control simultaneously the order of both and . However, Lenstra’s methods may be adapted to this more general setting of two multiplicative subgroups, the details of the proofs being given in [15]. We thus have analogues of Lenstra’s results in this setting.
Similarly as in Section 4, define
and
where is the smallest power of not dividing , and let (respectively ) denote the compositum of (respectively ) over the primes . Now if and only if the Artin symbol of with respect to belongs to a suitable conjugacy class of (namely the one fixing
and mapping ) and does not split completely in any of and for any prime . If is defined as the local density of primes satisfying this condition for all , by the analogue of Lenstra’s result (4.3) it suffices to show that the local densities for all in order to have .
Proceeding as in Section 4, one has to check two conditions. The first condition is , i.e. that . To do this one notes that the largest Abelian subfield of does not intersect nontrivially, and thus , implying that
(10.1) |
It follows that .
The second condition to check is that for any prime and squarefree we have
where
We do this by showing, more generally, that for any positive integers and satisfying and we have
(10.2) |
(cf. equation (4.5)). The upper bound implicit in the result is easy to establish. For the lower bound, apply the result in [10] to the numbers of the form over . We want to check that
for such and implies that . Similarly to the proof of (4.5), we obtain the divisibility relations
from which the result follows.
Having established we now proceed to proving that is independent of (for ). By a variant of Lenstra’s product formula in (4.3) for multiple variables one has
where is the number of conjugacy classes of fixing at least one of and . Since the terms of the infinite product do not depend on , it suffices to check that the local densities do not depend on .
Clearly for any prime and not divisible by we have , as the condition guarantees that does not split completely in and thus not in or . We may therefore assume that .
Similarly as one proves via (10.1), one obtains
for any , where
We thus see that there is exactly one element which fixes and whose restriction to belongs to . This leads to the following analogue of (4.4):
To conclude the proof one proves an analogue of (4.5) and (10.2). This time one claims that
when and . As before, the implied upper bound is easy. For the lower bound, we once again apply [10] and consider the numbers of the form
where and . Assume that such a number belongs to a cyclotomic field. Define
The integer must be a perfect power of order , and so
(10.3) |
and
(10.4) |
The rest is elementary. As , the first condition gives us
and similarly from the second condition
We now have, using , the relation and thus . This implies . Plugging this into (10.3) and (10.4) gives and , and thus .
Thus all of the results of Section 4 work with this new definition of . When it comes to Section 5, we redefine Pr as the uniform probability measure on . We also redefine
The second moment method (Lemma 3.1) gives an upper bound
for the probability that is prime with . By the Siegel–Walfisz theorem and the fact that implies , we see that the previous expression is
(10.5) |
The denominator here is
(10.6) |
Since fluctuates on the primes, we cannot a priori compute this sum just based on the value of . However, using the following lemma we can compute (10.6) in terms of .
Lemma 10.1.
Let , , and let . Then for and , we have
(10.7) |
We remark that much stronger tail bounds are known in the case ; see [33] for instance. However, for us the -aspect is crucial, since in particular taking we can conclude that all but -proportion of primes satisfy , say.
Proof.
This is a standard application of the method of moments. For any , the left-hand side of (10.7) is
(10.8) |
For , we have
where is a multiplicative function defined by for and for . By Möbius inversion, we have , where is a multiplicative function defined by for and when or , .
Now the sum over in (10.8) becomes
(10.9) |
By the Brun–Titchmarsh inequality, the contribution of the terms to the sum is
By the mean value theorem for derivatives, we have for . This together with the inequality lets us bound the previous expression by
by the prime number theorem and the choice .
By Lemma 10.1, for all but -proportion of we have , and for any , for all but -proportion of we have . Thus, the expression (10.6) takes the form
which is the same quantity as in Section 5.
To deal with the numerator in (10.5), we again apply Lemma 10.1 to write it as
Then we split the sum according to the value of . We reduce to proving (5.3) and (5.4) (with the difference that the sum in the definition of only goes up to and the sum in only goes over ). Estimate (5.4) is proved precisely as in the case of Theorem 1.3, since the set in Section 4 contains the set defined in this section. Estimate (5.3) is proved similarly as in Section 6, since the argument there is just based on Lemmas 4.1 and 4.2 for which we have analogues in this case, with the modification of considering only with in (6.1). ∎
11 Necessary conditions for primality of shifted exponentials
One might be tempted to conjecture that for Question 1.1 to have a positive answer it would suffice to look at the case there, that is, to show that has no fixed prime divisor and that does not factor as a result of a polynomial identity. These conditions are however not sufficient, as demonstrated by the sequence . For even, factors as the difference of two squares, whereas for odd we have (even though has no fixed prime divisor and is irreducible for odd). More generally, we have the following construction that shows the necessity of the “for every there exist ” part of Question 1.1.
Proposition 11.1.
Let be a prime. Then there exist integers such that
-
(i)
The sequence has no fixed prime divisor;
-
(ii)
The polynomial is irreducible, where is the largest integer such that is an th power;
but for every either of the following holds:
-
(iii)
The sequence has a fixed prime divisor;
-
(iv)
The polynomial is reducible.
Proof.
By Dirichlet’s theorem, we can pick odd, distinct primes . Now the congruences all have a solution with , so by the Chinese remainder theorem we can find an even which is not a perfect power and which satisfies for all . Then pick such that for all (this can be done, since is solvable for any primitive root ). Since the set of suitable forms an arithmetic progression and since , we may additionally require that and . Now let .
Conditions (i) and (ii) are now obvious, as and . Moreover, the polynomial factorizes as the difference of two th powers, and for each we have for all . This means that also conditions (iii) and (iv) hold. ∎
We remark that it seems difficult to tell whether the conditions of Question 1.1 hold for a given sequence. One case where this is difficult is the case of Sierpinski numbers: it has been conjectured by Erdős that if is a Sierpinski number (that is, is composite for all natural numbers ), then the smallest prime divisor of is bounded (see [7, Conjecture 2]). This corresponds to condition (i) in Question 1.1 for , (since for large primes the congruence is solvable if and only if is solvable). In [7], Filaseta, Finch and Kozek state that it is “highly likely” that Erdős’s conjecture is false, as the conjecture does not take into account polynomial identities, corresponding to condition (ii) in Question 1.1. They conjecture that Erdős’s intuition is correct for all power-free . They also conjecture that the smallest prime divisor of is unbounded as varies. Our proof of Theorem 1.6 gives that along almost all natural numbers , but does not rule out the smallest prime factor being bounded.
References
- [1] P. Balister, B. Bollobás, R. Morris, J. Sahasrabudhe, and M. Tiba. On the Erdős covering problem: the density of the uncovered set. Invent. Math., 228(1):377–414, 2022.
- [2] J. Bourgain, A. Gamburd, and P. Sarnak. Affine linear sieve, expanders, and sum-product. Invent. Math., 179(3):559–644, 2010.
- [3] J. Bourgain, A. Gamburd, and P. Sarnak. Markoff triples and strong approximation. C. R. Math. Acad. Sci. Paris, 354(2):131–135, 2016.
- [4] G. Cooke and P. J. Weinberger. On the construction of division chains in algebraic number rings, with applications to . Comm. Algebra, 3:481–524, 1975.
- [5] K. Debaene. Explicit counting of ideals and a Brun-Titchmarsh inequality for the Chebotarev density theorem. Int. J. Number Theory, 15(5):883–905, 2019.
- [6] C. Elsholtz. Density of primes in sequences of the form . MathOverflow answer. https://mathoverflow.net/q/268957.
- [7] M. Filaseta, C. Finch, and M. Kozek. On powers associated with Sierpiński numbers, Riesel numbers and Polignac’s conjecture. Journal of Number Theory, 128(7):1916 – 1940, 2008.
- [8] M. Filaseta, K. Ford, S. Konyagin, C. Pomerance, and G. Yu. Sieving by large integers and covering systems of congruences. J. Amer. Math. Soc., 20(2):495–517, 2007.
- [9] P. X. Gallagher. The number of conjugacy classes in a finite group. In Representation theory of finite groups and related topics (Proc. Sympos. Pure Math., Vol. XXI, Univ. Wisconsin, Madison, Wis., 1970), pages 51–52, 1971.
- [10] P. Garrett. Linear independence of roots. http://www-users.math.umn.edu/~garrett/m/v/linear_indep_roots.pdf.
- [11] R. K. Guy. Unsolved problems in number theory. Problem Books in Mathematics. Springer-Verlag, New York, second edition, 1994. Unsolved Problems in Intuitive Mathematics, I.
- [12] C. Hooley. On Artin’s conjecture. J. Reine Angew. Math., 225:209–220, 1967.
- [13] C. Hooley. Applications of sieve methods to the theory of numbers. Cambridge University Press, Cambridge-New York-Melbourne, 1976. Cambridge Tracts in Mathematics, No. 70.
- [14] H. Iwaniec and E. Kowalski. Analytic number theory, volume 53 of American Mathematical Society Colloquium Publications. American Mathematical Society, Providence, RI, 2004.
- [15] O. Järviniemi. Equality of orders of a set of integers modulo a prime. Proc. Amer. Math. Soc., 149(9):3651–3668, 2021.
- [16] G. Kalai. Proposals for polymath projects. MathOverflow answer. https://mathoverflow.net/q/229580.
- [17] J. C. Lagarias and A. M. Odlyzko. Effective versions of the Chebotarev density theorem. In Algebraic number fields: -functions and Galois properties (Proc. Sympos., Univ. Durham, Durham, 1975), pages 409–464, 1977.
- [18] H. W. Lenstra, Jr. On Artin’s conjecture and Euclid’s algorithm in global fields. Invent. Math., 42:201–224, 1977.
- [19] H. L. Montgomery. The pair correlation of zeros of the zeta function. In Analytic number theory (Proc. Sympos. Pure Math., Vol. XXIV, St. Louis Univ., St. Louis, Mo., 1972), pages 181–193, 1973.
- [20] P. Moree and P. Stevenhagen. A two-variable Artin conjecture. J. Number Theory, 85(2):291–304, 2000.
- [21] M. R. Murty, V. K. Murty, and P.-J. Wong. The Chebotarev density theorem and the pair correlation conjecture. J. Ramanujan Math. Soc., 33(4):399–426, 2018.
- [22] M. R. Murty and A. Perelli. The pair correlation of zeros of functions in the Selberg class. International Mathematics Research Notices, 1999(10):531–545, 01 1999.
- [23] M. R. Murty, F. Séguin, and C. L. Stewart. A lower bound for the two-variable Artin conjecture and prime divisors of recurrence sequences. J. Number Theory, 194:8–29, 2019.
- [24] J. Neukirch. Algebraic number theory, volume 322 of Grundlehren der Mathematischen Wissenschaften [Fundamental Principles of Mathematical Sciences]. Springer-Verlag, Berlin, 1999. Translated from the 1992 German original and with a note by Norbert Schappacher, With a foreword by G. Harder.
- [25] A. M. Odlyzko. On the distribution of spacings between zeros of the zeta function. Math. Comp., 48(177):273–308, 1987.
- [26] G. J. Rieger. Über Primzahlen und dünne Folgen. Arch. Math. (Basel), 28(6):600–602, 1977.
- [27] M. Roitman. On Zsigmondy primes. Proc. Amer. Math. Soc., 125(7):1913–1919, 1997.
- [28] Z. Rudnick and P. Sarnak. Zeros of principal -functions and random matrix theory. Duke Math. J., 81(2):269–322, 1996. A celebration of John F. Nash, Jr.
- [29] A. Salehi Golsefidy and P. Sarnak. The affine sieve. J. Amer. Math. Soc., 26(4):1085–1105, 2013.
- [30] P. Sarnak. Slide presentation. http://geometrie.math.cnrs.fr/Sarnak.pdf.
- [31] J.-P. Serre. Quelques applications du théorème de densité de Chebotarev. Inst. Hautes Études Sci. Publ. Math., (54):323–401, 1981.
- [32] J. Thorner and A. Zaman. A Chebotarev Variant of the Brun–Titchmarsh Theorem and Bounds for the Lang-Trotter conjectures. International Mathematics Research Notices, 2018(16):4991–5027, 03 2017.
- [33] A. Weingartner. The distribution functions of and . Proc. Amer. Math. Soc., 135(9):2677–2681, 2007.
- [34] P.-J. Wong. Applications of group theory to conjectures of Artin and Langlands. Int. J. Number Theory, 14(3):881–898, 2018.